Condition in terms of b and a if $ax^2+bx+c=0$ has two consecutive odd positive integers as roots











up vote
2
down vote

favorite
2













The roots of the equation$$ax^2+bx+c=0$$, where $a geq 0$, are two consecutive odd positive integers, then




(A) $|b|leq 4a$



(B) $|b|geq 4a$



(C) $|b|=2a$



(D) None of these



My attempt



Let p and q be the roots then if they are consecutive positive integers (q>p) then $$ pq=frac{c}{a} geq 0$$
So, $$c geq 0$$ and $$q-p=2$$
So, $$frac{sqrt{b^2-4ac}}{a}=2$$
So,$$|b|>2a$$
$(Since, a>0,c>0)$



But I know that 4ac should be taken in consideration since its not equal to zero. But I don't know how to use it.



Any hints and suggestions are welcome!










share|cite|improve this question




























    up vote
    2
    down vote

    favorite
    2













    The roots of the equation$$ax^2+bx+c=0$$, where $a geq 0$, are two consecutive odd positive integers, then




    (A) $|b|leq 4a$



    (B) $|b|geq 4a$



    (C) $|b|=2a$



    (D) None of these



    My attempt



    Let p and q be the roots then if they are consecutive positive integers (q>p) then $$ pq=frac{c}{a} geq 0$$
    So, $$c geq 0$$ and $$q-p=2$$
    So, $$frac{sqrt{b^2-4ac}}{a}=2$$
    So,$$|b|>2a$$
    $(Since, a>0,c>0)$



    But I know that 4ac should be taken in consideration since its not equal to zero. But I don't know how to use it.



    Any hints and suggestions are welcome!










    share|cite|improve this question


























      up vote
      2
      down vote

      favorite
      2









      up vote
      2
      down vote

      favorite
      2






      2






      The roots of the equation$$ax^2+bx+c=0$$, where $a geq 0$, are two consecutive odd positive integers, then




      (A) $|b|leq 4a$



      (B) $|b|geq 4a$



      (C) $|b|=2a$



      (D) None of these



      My attempt



      Let p and q be the roots then if they are consecutive positive integers (q>p) then $$ pq=frac{c}{a} geq 0$$
      So, $$c geq 0$$ and $$q-p=2$$
      So, $$frac{sqrt{b^2-4ac}}{a}=2$$
      So,$$|b|>2a$$
      $(Since, a>0,c>0)$



      But I know that 4ac should be taken in consideration since its not equal to zero. But I don't know how to use it.



      Any hints and suggestions are welcome!










      share|cite|improve this question
















      The roots of the equation$$ax^2+bx+c=0$$, where $a geq 0$, are two consecutive odd positive integers, then




      (A) $|b|leq 4a$



      (B) $|b|geq 4a$



      (C) $|b|=2a$



      (D) None of these



      My attempt



      Let p and q be the roots then if they are consecutive positive integers (q>p) then $$ pq=frac{c}{a} geq 0$$
      So, $$c geq 0$$ and $$q-p=2$$
      So, $$frac{sqrt{b^2-4ac}}{a}=2$$
      So,$$|b|>2a$$
      $(Since, a>0,c>0)$



      But I know that 4ac should be taken in consideration since its not equal to zero. But I don't know how to use it.



      Any hints and suggestions are welcome!







      algebra-precalculus quadratics self-learning






      share|cite|improve this question















      share|cite|improve this question













      share|cite|improve this question




      share|cite|improve this question








      edited Nov 21 at 21:34









      Micah

      29.5k1363104




      29.5k1363104










      asked Nov 21 at 21:20









      jayant98

      35414




      35414






















          4 Answers
          4






          active

          oldest

          votes

















          up vote
          5
          down vote



          accepted










          Simply, If $2n-1$ and $2n+1$ are the roots (with $nge 1$) then
          $$ax^2+bx+c=a(x-2n+1)(x-2n-1)=a((x-2n)^2-1)=a(x^2-4nx+4n^2-1) $$
          so $b=-4na$ and hence $|b|=4nage 4a$.






          share|cite|improve this answer





















          • Oh, you have the easier and more simple way! Thanks.
            – jayant98
            Nov 21 at 21:34


















          up vote
          1
          down vote













          Using the quadratic formula, we have that (using $p<q$ as the roots):
          $$p+2=q$$
          $$frac{-b-sqrt{b^2-4ac}}{2a}+2=frac{-b+sqrt{b^2-4ac}}{2a}$$
          $$frac{-b}{2a}-frac{sqrt{b^2-4ac}}{2a}+2=frac{-b}{2a}+frac{sqrt{b^2-4ac}}{2a}$$
          $$2=2frac{sqrt{b^2-4ac}}{2a}$$
          $$2a=sqrt{b^2-4ac}$$
          $$4a^2=b^2-4ac$$
          $$b^2=4a^2+4ac$$
          $$frac{b^2}{a^2}=4(1+frac{c}{a})=4(1+pq)$$
          $$frac{|b|}{a}=2sqrt{1+pq}$$
          Then, given that $p,q$ are different odd integers, you can show that $pqgeq3$, so that $2sqrt{1+pq}geq2sqrt{1+3}=4$






          share|cite|improve this answer




























            up vote
            1
            down vote













            Since we are given that there are two distinct roots and that $a ge 0$, we must have



            $a > 0, tag 0$



            since otherwise (i.e., with $a = 0$), the "quadratic" $bx + c$ has at most one root.



            Let



            $n ge 0, tag 1$



            $r = 2n + 1, tag 2$



            $s = 2n + 3; tag 3$



            suppose for the moment



            $a = 1; tag 4$



            then



            $(x - r)(x - s) = (x - (2n + 1))(x - (2n + 3)) = x^2 -(4n + 4)x + (2n + 1)(2n + 3) = x^2 -(4n + 4)x + (4n^2 + 8n + 3) = 0; tag 5$



            here we have



            $b = -(4n + 4), tag 6$



            whence



            $vert b vert = 4n + 4 ge 4 = 4a; tag 7$



            thus (B) binds when $a = 1$; now if



            $a ne 1, tag 8$



            the quadratic of the form



            $ax^2 + bx + c = a(x^2 + dfrac{b}{a} x + dfrac{c}{a}) tag 9$



            has zeroes $2n + 1$, $2n + 3$ provided



            $(x - (2n + 1))(x - (2n + 3)) = x^2 -(4n + 4)x + (4n^2 + 8n + 3) = x^2 + dfrac{b}{a} x + dfrac{c}{a}; tag{10}$



            thus,



            $dfrac{b}{a} = -(4n + 4), tag{11}$



            or



            $b = -(4n + 4)a, tag{12}$



            whence, with $a > 0$,



            $vert b vert = (4n + 4)a ge 4a, tag{13}$



            and we see that the correct choice is (B) here as well.






            share|cite|improve this answer




























              up vote
              0
              down vote













              Easy way to check:



              Take a quadratic equation which has 1 and 3 as roots. $(x-1)(x-3)=x^2-4x+4x$, so you see that it is possible for $|b|=4|a|$, ruling out choice (C).



              Do the problem again choosing the roots 3 and 5. $(x-3)(x-5)=x^2-8x+15$, so you can conclude that $|b|>=4a$, and (B) is correct. (If you continue choosing larger pairs of numbers as roots, $frac{|b|}{a}$ will only increase.)






              share|cite|improve this answer





















              • Sorry @MoKo19 but we have to give answer in subjective way...So we can't just do that...This question given just has the options...but we have to solve it 'properly'.
                – jayant98
                Nov 21 at 21:32










              • @jayant98: Not sure what you mean by 'properly'. If you mean analytically, then generate the quadratic equation where $a=1$ with roots $p,p+2$. Because This will give you $y=(x-p)(x-p-2)=x^2-2px-2x+p^2+2p$, and $frac{|b|}{a}=|b|=|2p+2|=2p+2$. For odd integer roots, $p=1,3,5,...$, so the ratio will be $4,8,12,16,...$. I will write a different answer showing how to solve via the quadratic formula.
                – MoKo19
                Nov 21 at 21:42











              Your Answer





              StackExchange.ifUsing("editor", function () {
              return StackExchange.using("mathjaxEditing", function () {
              StackExchange.MarkdownEditor.creationCallbacks.add(function (editor, postfix) {
              StackExchange.mathjaxEditing.prepareWmdForMathJax(editor, postfix, [["$", "$"], ["\\(","\\)"]]);
              });
              });
              }, "mathjax-editing");

              StackExchange.ready(function() {
              var channelOptions = {
              tags: "".split(" "),
              id: "69"
              };
              initTagRenderer("".split(" "), "".split(" "), channelOptions);

              StackExchange.using("externalEditor", function() {
              // Have to fire editor after snippets, if snippets enabled
              if (StackExchange.settings.snippets.snippetsEnabled) {
              StackExchange.using("snippets", function() {
              createEditor();
              });
              }
              else {
              createEditor();
              }
              });

              function createEditor() {
              StackExchange.prepareEditor({
              heartbeatType: 'answer',
              convertImagesToLinks: true,
              noModals: true,
              showLowRepImageUploadWarning: true,
              reputationToPostImages: 10,
              bindNavPrevention: true,
              postfix: "",
              imageUploader: {
              brandingHtml: "Powered by u003ca class="icon-imgur-white" href="https://imgur.com/"u003eu003c/au003e",
              contentPolicyHtml: "User contributions licensed under u003ca href="https://creativecommons.org/licenses/by-sa/3.0/"u003ecc by-sa 3.0 with attribution requiredu003c/au003e u003ca href="https://stackoverflow.com/legal/content-policy"u003e(content policy)u003c/au003e",
              allowUrls: true
              },
              noCode: true, onDemand: true,
              discardSelector: ".discard-answer"
              ,immediatelyShowMarkdownHelp:true
              });


              }
              });














              draft saved

              draft discarded


















              StackExchange.ready(
              function () {
              StackExchange.openid.initPostLogin('.new-post-login', 'https%3a%2f%2fmath.stackexchange.com%2fquestions%2f3008382%2fcondition-in-terms-of-b-and-a-if-ax2bxc-0-has-two-consecutive-odd-positive%23new-answer', 'question_page');
              }
              );

              Post as a guest















              Required, but never shown

























              4 Answers
              4






              active

              oldest

              votes








              4 Answers
              4






              active

              oldest

              votes









              active

              oldest

              votes






              active

              oldest

              votes








              up vote
              5
              down vote



              accepted










              Simply, If $2n-1$ and $2n+1$ are the roots (with $nge 1$) then
              $$ax^2+bx+c=a(x-2n+1)(x-2n-1)=a((x-2n)^2-1)=a(x^2-4nx+4n^2-1) $$
              so $b=-4na$ and hence $|b|=4nage 4a$.






              share|cite|improve this answer





















              • Oh, you have the easier and more simple way! Thanks.
                – jayant98
                Nov 21 at 21:34















              up vote
              5
              down vote



              accepted










              Simply, If $2n-1$ and $2n+1$ are the roots (with $nge 1$) then
              $$ax^2+bx+c=a(x-2n+1)(x-2n-1)=a((x-2n)^2-1)=a(x^2-4nx+4n^2-1) $$
              so $b=-4na$ and hence $|b|=4nage 4a$.






              share|cite|improve this answer





















              • Oh, you have the easier and more simple way! Thanks.
                – jayant98
                Nov 21 at 21:34













              up vote
              5
              down vote



              accepted







              up vote
              5
              down vote



              accepted






              Simply, If $2n-1$ and $2n+1$ are the roots (with $nge 1$) then
              $$ax^2+bx+c=a(x-2n+1)(x-2n-1)=a((x-2n)^2-1)=a(x^2-4nx+4n^2-1) $$
              so $b=-4na$ and hence $|b|=4nage 4a$.






              share|cite|improve this answer












              Simply, If $2n-1$ and $2n+1$ are the roots (with $nge 1$) then
              $$ax^2+bx+c=a(x-2n+1)(x-2n-1)=a((x-2n)^2-1)=a(x^2-4nx+4n^2-1) $$
              so $b=-4na$ and hence $|b|=4nage 4a$.







              share|cite|improve this answer












              share|cite|improve this answer



              share|cite|improve this answer










              answered Nov 21 at 21:33









              Hagen von Eitzen

              275k21268495




              275k21268495












              • Oh, you have the easier and more simple way! Thanks.
                – jayant98
                Nov 21 at 21:34


















              • Oh, you have the easier and more simple way! Thanks.
                – jayant98
                Nov 21 at 21:34
















              Oh, you have the easier and more simple way! Thanks.
              – jayant98
              Nov 21 at 21:34




              Oh, you have the easier and more simple way! Thanks.
              – jayant98
              Nov 21 at 21:34










              up vote
              1
              down vote













              Using the quadratic formula, we have that (using $p<q$ as the roots):
              $$p+2=q$$
              $$frac{-b-sqrt{b^2-4ac}}{2a}+2=frac{-b+sqrt{b^2-4ac}}{2a}$$
              $$frac{-b}{2a}-frac{sqrt{b^2-4ac}}{2a}+2=frac{-b}{2a}+frac{sqrt{b^2-4ac}}{2a}$$
              $$2=2frac{sqrt{b^2-4ac}}{2a}$$
              $$2a=sqrt{b^2-4ac}$$
              $$4a^2=b^2-4ac$$
              $$b^2=4a^2+4ac$$
              $$frac{b^2}{a^2}=4(1+frac{c}{a})=4(1+pq)$$
              $$frac{|b|}{a}=2sqrt{1+pq}$$
              Then, given that $p,q$ are different odd integers, you can show that $pqgeq3$, so that $2sqrt{1+pq}geq2sqrt{1+3}=4$






              share|cite|improve this answer

























                up vote
                1
                down vote













                Using the quadratic formula, we have that (using $p<q$ as the roots):
                $$p+2=q$$
                $$frac{-b-sqrt{b^2-4ac}}{2a}+2=frac{-b+sqrt{b^2-4ac}}{2a}$$
                $$frac{-b}{2a}-frac{sqrt{b^2-4ac}}{2a}+2=frac{-b}{2a}+frac{sqrt{b^2-4ac}}{2a}$$
                $$2=2frac{sqrt{b^2-4ac}}{2a}$$
                $$2a=sqrt{b^2-4ac}$$
                $$4a^2=b^2-4ac$$
                $$b^2=4a^2+4ac$$
                $$frac{b^2}{a^2}=4(1+frac{c}{a})=4(1+pq)$$
                $$frac{|b|}{a}=2sqrt{1+pq}$$
                Then, given that $p,q$ are different odd integers, you can show that $pqgeq3$, so that $2sqrt{1+pq}geq2sqrt{1+3}=4$






                share|cite|improve this answer























                  up vote
                  1
                  down vote










                  up vote
                  1
                  down vote









                  Using the quadratic formula, we have that (using $p<q$ as the roots):
                  $$p+2=q$$
                  $$frac{-b-sqrt{b^2-4ac}}{2a}+2=frac{-b+sqrt{b^2-4ac}}{2a}$$
                  $$frac{-b}{2a}-frac{sqrt{b^2-4ac}}{2a}+2=frac{-b}{2a}+frac{sqrt{b^2-4ac}}{2a}$$
                  $$2=2frac{sqrt{b^2-4ac}}{2a}$$
                  $$2a=sqrt{b^2-4ac}$$
                  $$4a^2=b^2-4ac$$
                  $$b^2=4a^2+4ac$$
                  $$frac{b^2}{a^2}=4(1+frac{c}{a})=4(1+pq)$$
                  $$frac{|b|}{a}=2sqrt{1+pq}$$
                  Then, given that $p,q$ are different odd integers, you can show that $pqgeq3$, so that $2sqrt{1+pq}geq2sqrt{1+3}=4$






                  share|cite|improve this answer












                  Using the quadratic formula, we have that (using $p<q$ as the roots):
                  $$p+2=q$$
                  $$frac{-b-sqrt{b^2-4ac}}{2a}+2=frac{-b+sqrt{b^2-4ac}}{2a}$$
                  $$frac{-b}{2a}-frac{sqrt{b^2-4ac}}{2a}+2=frac{-b}{2a}+frac{sqrt{b^2-4ac}}{2a}$$
                  $$2=2frac{sqrt{b^2-4ac}}{2a}$$
                  $$2a=sqrt{b^2-4ac}$$
                  $$4a^2=b^2-4ac$$
                  $$b^2=4a^2+4ac$$
                  $$frac{b^2}{a^2}=4(1+frac{c}{a})=4(1+pq)$$
                  $$frac{|b|}{a}=2sqrt{1+pq}$$
                  Then, given that $p,q$ are different odd integers, you can show that $pqgeq3$, so that $2sqrt{1+pq}geq2sqrt{1+3}=4$







                  share|cite|improve this answer












                  share|cite|improve this answer



                  share|cite|improve this answer










                  answered Nov 21 at 21:57









                  MoKo19

                  1914




                  1914






















                      up vote
                      1
                      down vote













                      Since we are given that there are two distinct roots and that $a ge 0$, we must have



                      $a > 0, tag 0$



                      since otherwise (i.e., with $a = 0$), the "quadratic" $bx + c$ has at most one root.



                      Let



                      $n ge 0, tag 1$



                      $r = 2n + 1, tag 2$



                      $s = 2n + 3; tag 3$



                      suppose for the moment



                      $a = 1; tag 4$



                      then



                      $(x - r)(x - s) = (x - (2n + 1))(x - (2n + 3)) = x^2 -(4n + 4)x + (2n + 1)(2n + 3) = x^2 -(4n + 4)x + (4n^2 + 8n + 3) = 0; tag 5$



                      here we have



                      $b = -(4n + 4), tag 6$



                      whence



                      $vert b vert = 4n + 4 ge 4 = 4a; tag 7$



                      thus (B) binds when $a = 1$; now if



                      $a ne 1, tag 8$



                      the quadratic of the form



                      $ax^2 + bx + c = a(x^2 + dfrac{b}{a} x + dfrac{c}{a}) tag 9$



                      has zeroes $2n + 1$, $2n + 3$ provided



                      $(x - (2n + 1))(x - (2n + 3)) = x^2 -(4n + 4)x + (4n^2 + 8n + 3) = x^2 + dfrac{b}{a} x + dfrac{c}{a}; tag{10}$



                      thus,



                      $dfrac{b}{a} = -(4n + 4), tag{11}$



                      or



                      $b = -(4n + 4)a, tag{12}$



                      whence, with $a > 0$,



                      $vert b vert = (4n + 4)a ge 4a, tag{13}$



                      and we see that the correct choice is (B) here as well.






                      share|cite|improve this answer

























                        up vote
                        1
                        down vote













                        Since we are given that there are two distinct roots and that $a ge 0$, we must have



                        $a > 0, tag 0$



                        since otherwise (i.e., with $a = 0$), the "quadratic" $bx + c$ has at most one root.



                        Let



                        $n ge 0, tag 1$



                        $r = 2n + 1, tag 2$



                        $s = 2n + 3; tag 3$



                        suppose for the moment



                        $a = 1; tag 4$



                        then



                        $(x - r)(x - s) = (x - (2n + 1))(x - (2n + 3)) = x^2 -(4n + 4)x + (2n + 1)(2n + 3) = x^2 -(4n + 4)x + (4n^2 + 8n + 3) = 0; tag 5$



                        here we have



                        $b = -(4n + 4), tag 6$



                        whence



                        $vert b vert = 4n + 4 ge 4 = 4a; tag 7$



                        thus (B) binds when $a = 1$; now if



                        $a ne 1, tag 8$



                        the quadratic of the form



                        $ax^2 + bx + c = a(x^2 + dfrac{b}{a} x + dfrac{c}{a}) tag 9$



                        has zeroes $2n + 1$, $2n + 3$ provided



                        $(x - (2n + 1))(x - (2n + 3)) = x^2 -(4n + 4)x + (4n^2 + 8n + 3) = x^2 + dfrac{b}{a} x + dfrac{c}{a}; tag{10}$



                        thus,



                        $dfrac{b}{a} = -(4n + 4), tag{11}$



                        or



                        $b = -(4n + 4)a, tag{12}$



                        whence, with $a > 0$,



                        $vert b vert = (4n + 4)a ge 4a, tag{13}$



                        and we see that the correct choice is (B) here as well.






                        share|cite|improve this answer























                          up vote
                          1
                          down vote










                          up vote
                          1
                          down vote









                          Since we are given that there are two distinct roots and that $a ge 0$, we must have



                          $a > 0, tag 0$



                          since otherwise (i.e., with $a = 0$), the "quadratic" $bx + c$ has at most one root.



                          Let



                          $n ge 0, tag 1$



                          $r = 2n + 1, tag 2$



                          $s = 2n + 3; tag 3$



                          suppose for the moment



                          $a = 1; tag 4$



                          then



                          $(x - r)(x - s) = (x - (2n + 1))(x - (2n + 3)) = x^2 -(4n + 4)x + (2n + 1)(2n + 3) = x^2 -(4n + 4)x + (4n^2 + 8n + 3) = 0; tag 5$



                          here we have



                          $b = -(4n + 4), tag 6$



                          whence



                          $vert b vert = 4n + 4 ge 4 = 4a; tag 7$



                          thus (B) binds when $a = 1$; now if



                          $a ne 1, tag 8$



                          the quadratic of the form



                          $ax^2 + bx + c = a(x^2 + dfrac{b}{a} x + dfrac{c}{a}) tag 9$



                          has zeroes $2n + 1$, $2n + 3$ provided



                          $(x - (2n + 1))(x - (2n + 3)) = x^2 -(4n + 4)x + (4n^2 + 8n + 3) = x^2 + dfrac{b}{a} x + dfrac{c}{a}; tag{10}$



                          thus,



                          $dfrac{b}{a} = -(4n + 4), tag{11}$



                          or



                          $b = -(4n + 4)a, tag{12}$



                          whence, with $a > 0$,



                          $vert b vert = (4n + 4)a ge 4a, tag{13}$



                          and we see that the correct choice is (B) here as well.






                          share|cite|improve this answer












                          Since we are given that there are two distinct roots and that $a ge 0$, we must have



                          $a > 0, tag 0$



                          since otherwise (i.e., with $a = 0$), the "quadratic" $bx + c$ has at most one root.



                          Let



                          $n ge 0, tag 1$



                          $r = 2n + 1, tag 2$



                          $s = 2n + 3; tag 3$



                          suppose for the moment



                          $a = 1; tag 4$



                          then



                          $(x - r)(x - s) = (x - (2n + 1))(x - (2n + 3)) = x^2 -(4n + 4)x + (2n + 1)(2n + 3) = x^2 -(4n + 4)x + (4n^2 + 8n + 3) = 0; tag 5$



                          here we have



                          $b = -(4n + 4), tag 6$



                          whence



                          $vert b vert = 4n + 4 ge 4 = 4a; tag 7$



                          thus (B) binds when $a = 1$; now if



                          $a ne 1, tag 8$



                          the quadratic of the form



                          $ax^2 + bx + c = a(x^2 + dfrac{b}{a} x + dfrac{c}{a}) tag 9$



                          has zeroes $2n + 1$, $2n + 3$ provided



                          $(x - (2n + 1))(x - (2n + 3)) = x^2 -(4n + 4)x + (4n^2 + 8n + 3) = x^2 + dfrac{b}{a} x + dfrac{c}{a}; tag{10}$



                          thus,



                          $dfrac{b}{a} = -(4n + 4), tag{11}$



                          or



                          $b = -(4n + 4)a, tag{12}$



                          whence, with $a > 0$,



                          $vert b vert = (4n + 4)a ge 4a, tag{13}$



                          and we see that the correct choice is (B) here as well.







                          share|cite|improve this answer












                          share|cite|improve this answer



                          share|cite|improve this answer










                          answered Nov 22 at 8:41









                          Robert Lewis

                          42.7k22862




                          42.7k22862






















                              up vote
                              0
                              down vote













                              Easy way to check:



                              Take a quadratic equation which has 1 and 3 as roots. $(x-1)(x-3)=x^2-4x+4x$, so you see that it is possible for $|b|=4|a|$, ruling out choice (C).



                              Do the problem again choosing the roots 3 and 5. $(x-3)(x-5)=x^2-8x+15$, so you can conclude that $|b|>=4a$, and (B) is correct. (If you continue choosing larger pairs of numbers as roots, $frac{|b|}{a}$ will only increase.)






                              share|cite|improve this answer





















                              • Sorry @MoKo19 but we have to give answer in subjective way...So we can't just do that...This question given just has the options...but we have to solve it 'properly'.
                                – jayant98
                                Nov 21 at 21:32










                              • @jayant98: Not sure what you mean by 'properly'. If you mean analytically, then generate the quadratic equation where $a=1$ with roots $p,p+2$. Because This will give you $y=(x-p)(x-p-2)=x^2-2px-2x+p^2+2p$, and $frac{|b|}{a}=|b|=|2p+2|=2p+2$. For odd integer roots, $p=1,3,5,...$, so the ratio will be $4,8,12,16,...$. I will write a different answer showing how to solve via the quadratic formula.
                                – MoKo19
                                Nov 21 at 21:42















                              up vote
                              0
                              down vote













                              Easy way to check:



                              Take a quadratic equation which has 1 and 3 as roots. $(x-1)(x-3)=x^2-4x+4x$, so you see that it is possible for $|b|=4|a|$, ruling out choice (C).



                              Do the problem again choosing the roots 3 and 5. $(x-3)(x-5)=x^2-8x+15$, so you can conclude that $|b|>=4a$, and (B) is correct. (If you continue choosing larger pairs of numbers as roots, $frac{|b|}{a}$ will only increase.)






                              share|cite|improve this answer





















                              • Sorry @MoKo19 but we have to give answer in subjective way...So we can't just do that...This question given just has the options...but we have to solve it 'properly'.
                                – jayant98
                                Nov 21 at 21:32










                              • @jayant98: Not sure what you mean by 'properly'. If you mean analytically, then generate the quadratic equation where $a=1$ with roots $p,p+2$. Because This will give you $y=(x-p)(x-p-2)=x^2-2px-2x+p^2+2p$, and $frac{|b|}{a}=|b|=|2p+2|=2p+2$. For odd integer roots, $p=1,3,5,...$, so the ratio will be $4,8,12,16,...$. I will write a different answer showing how to solve via the quadratic formula.
                                – MoKo19
                                Nov 21 at 21:42













                              up vote
                              0
                              down vote










                              up vote
                              0
                              down vote









                              Easy way to check:



                              Take a quadratic equation which has 1 and 3 as roots. $(x-1)(x-3)=x^2-4x+4x$, so you see that it is possible for $|b|=4|a|$, ruling out choice (C).



                              Do the problem again choosing the roots 3 and 5. $(x-3)(x-5)=x^2-8x+15$, so you can conclude that $|b|>=4a$, and (B) is correct. (If you continue choosing larger pairs of numbers as roots, $frac{|b|}{a}$ will only increase.)






                              share|cite|improve this answer












                              Easy way to check:



                              Take a quadratic equation which has 1 and 3 as roots. $(x-1)(x-3)=x^2-4x+4x$, so you see that it is possible for $|b|=4|a|$, ruling out choice (C).



                              Do the problem again choosing the roots 3 and 5. $(x-3)(x-5)=x^2-8x+15$, so you can conclude that $|b|>=4a$, and (B) is correct. (If you continue choosing larger pairs of numbers as roots, $frac{|b|}{a}$ will only increase.)







                              share|cite|improve this answer












                              share|cite|improve this answer



                              share|cite|improve this answer










                              answered Nov 21 at 21:29









                              MoKo19

                              1914




                              1914












                              • Sorry @MoKo19 but we have to give answer in subjective way...So we can't just do that...This question given just has the options...but we have to solve it 'properly'.
                                – jayant98
                                Nov 21 at 21:32










                              • @jayant98: Not sure what you mean by 'properly'. If you mean analytically, then generate the quadratic equation where $a=1$ with roots $p,p+2$. Because This will give you $y=(x-p)(x-p-2)=x^2-2px-2x+p^2+2p$, and $frac{|b|}{a}=|b|=|2p+2|=2p+2$. For odd integer roots, $p=1,3,5,...$, so the ratio will be $4,8,12,16,...$. I will write a different answer showing how to solve via the quadratic formula.
                                – MoKo19
                                Nov 21 at 21:42


















                              • Sorry @MoKo19 but we have to give answer in subjective way...So we can't just do that...This question given just has the options...but we have to solve it 'properly'.
                                – jayant98
                                Nov 21 at 21:32










                              • @jayant98: Not sure what you mean by 'properly'. If you mean analytically, then generate the quadratic equation where $a=1$ with roots $p,p+2$. Because This will give you $y=(x-p)(x-p-2)=x^2-2px-2x+p^2+2p$, and $frac{|b|}{a}=|b|=|2p+2|=2p+2$. For odd integer roots, $p=1,3,5,...$, so the ratio will be $4,8,12,16,...$. I will write a different answer showing how to solve via the quadratic formula.
                                – MoKo19
                                Nov 21 at 21:42
















                              Sorry @MoKo19 but we have to give answer in subjective way...So we can't just do that...This question given just has the options...but we have to solve it 'properly'.
                              – jayant98
                              Nov 21 at 21:32




                              Sorry @MoKo19 but we have to give answer in subjective way...So we can't just do that...This question given just has the options...but we have to solve it 'properly'.
                              – jayant98
                              Nov 21 at 21:32












                              @jayant98: Not sure what you mean by 'properly'. If you mean analytically, then generate the quadratic equation where $a=1$ with roots $p,p+2$. Because This will give you $y=(x-p)(x-p-2)=x^2-2px-2x+p^2+2p$, and $frac{|b|}{a}=|b|=|2p+2|=2p+2$. For odd integer roots, $p=1,3,5,...$, so the ratio will be $4,8,12,16,...$. I will write a different answer showing how to solve via the quadratic formula.
                              – MoKo19
                              Nov 21 at 21:42




                              @jayant98: Not sure what you mean by 'properly'. If you mean analytically, then generate the quadratic equation where $a=1$ with roots $p,p+2$. Because This will give you $y=(x-p)(x-p-2)=x^2-2px-2x+p^2+2p$, and $frac{|b|}{a}=|b|=|2p+2|=2p+2$. For odd integer roots, $p=1,3,5,...$, so the ratio will be $4,8,12,16,...$. I will write a different answer showing how to solve via the quadratic formula.
                              – MoKo19
                              Nov 21 at 21:42


















                              draft saved

                              draft discarded




















































                              Thanks for contributing an answer to Mathematics Stack Exchange!


                              • Please be sure to answer the question. Provide details and share your research!

                              But avoid



                              • Asking for help, clarification, or responding to other answers.

                              • Making statements based on opinion; back them up with references or personal experience.


                              Use MathJax to format equations. MathJax reference.


                              To learn more, see our tips on writing great answers.





                              Some of your past answers have not been well-received, and you're in danger of being blocked from answering.


                              Please pay close attention to the following guidance:


                              • Please be sure to answer the question. Provide details and share your research!

                              But avoid



                              • Asking for help, clarification, or responding to other answers.

                              • Making statements based on opinion; back them up with references or personal experience.


                              To learn more, see our tips on writing great answers.




                              draft saved


                              draft discarded














                              StackExchange.ready(
                              function () {
                              StackExchange.openid.initPostLogin('.new-post-login', 'https%3a%2f%2fmath.stackexchange.com%2fquestions%2f3008382%2fcondition-in-terms-of-b-and-a-if-ax2bxc-0-has-two-consecutive-odd-positive%23new-answer', 'question_page');
                              }
                              );

                              Post as a guest















                              Required, but never shown





















































                              Required, but never shown














                              Required, but never shown












                              Required, but never shown







                              Required, but never shown

































                              Required, but never shown














                              Required, but never shown












                              Required, but never shown







                              Required, but never shown







                              Popular posts from this blog

                              Quarter-circle Tiles

                              build a pushdown automaton that recognizes the reverse language of a given pushdown automaton?

                              Mont Emei